Difference between revisions of "1988 AHSME Problems/Problem 28"

(Created page with "==Problem== An unfair coin has probability <math>p</math> of coming up heads on a single toss. Let <math>w</math> be the probability that, in <math>5</math> independent toss of...")
 
 
Line 13: Line 13:
  
 
==Solution==
 
==Solution==
 
+
We have <math>w = {5\choose3}p^{3}(1-p)^{2} = 10p^{3}(1-p)^{2}</math>, so we need to solve <math>10p^{3}(1-p)^{2} = \frac{144}{625} \implies p^{3}(1-p)^{2} = \frac{72}{3125}</math>. Now observe that when <math>p=0</math>, the left-hand side evaluates to <math>0</math>, which is less than <math>\frac{72}{3125}</math>; when <math>p=\frac{1}{2}</math>, it evaluates to <math>\frac{1}{32}</math>, which is more than <math>\frac{72}{3125}</math>, and when <math>p=1</math>, it evaluates to <math>0</math> again. Thus, since <math>p^{3}(1-p)^{2}</math> is continuous, the Intermediate Value Theorem tells us there is a solution between <math>0</math> and <math>\frac{1}{2}</math> and another solution between <math>\frac{1}{2}</math> and <math>1</math>, meaning that there is not a unique value of <math>p</math>, so the answer is <math>\boxed{\text{D}}</math>.
  
  

Latest revision as of 14:50, 27 February 2018

Problem

An unfair coin has probability $p$ of coming up heads on a single toss. Let $w$ be the probability that, in $5$ independent toss of this coin, heads come up exactly $3$ times. If $w = 144 / 625$, then

$\textbf{(A)}\ p\text{ must be }\tfrac{2}{5}\qquad \textbf{(B)}\ p\text{ must be }\tfrac{3}{5}\qquad\\ \textbf{(C)}\ p\text{ must be greater than }\tfrac{3}{5}\qquad \textbf{(D)}\ p\text{ is not uniquely determined}\qquad\\ \textbf{(E)}\ \text{there is no value of } p \text{ for which }w =\tfrac{144}{625}$


Solution

We have $w = {5\choose3}p^{3}(1-p)^{2} = 10p^{3}(1-p)^{2}$, so we need to solve $10p^{3}(1-p)^{2} = \frac{144}{625} \implies p^{3}(1-p)^{2} = \frac{72}{3125}$. Now observe that when $p=0$, the left-hand side evaluates to $0$, which is less than $\frac{72}{3125}$; when $p=\frac{1}{2}$, it evaluates to $\frac{1}{32}$, which is more than $\frac{72}{3125}$, and when $p=1$, it evaluates to $0$ again. Thus, since $p^{3}(1-p)^{2}$ is continuous, the Intermediate Value Theorem tells us there is a solution between $0$ and $\frac{1}{2}$ and another solution between $\frac{1}{2}$ and $1$, meaning that there is not a unique value of $p$, so the answer is $\boxed{\text{D}}$.


See also

1988 AHSME (ProblemsAnswer KeyResources)
Preceded by
Problem 27
Followed by
Problem 29
1 2 3 4 5 6 7 8 9 10 11 12 13 14 15 16 17 18 19 20 21 22 23 24 25 26 27 28 29 30
All AHSME Problems and Solutions

The problems on this page are copyrighted by the Mathematical Association of America's American Mathematics Competitions. AMC logo.png